Đến nội dung

Hình ảnh

Topic ôn thi HSG lớp 10 Đồng Bằng Bắc Bộ và Olympic 30-4


  • Please log in to reply
Chủ đề này có 54 trả lời

#21
deathavailable

deathavailable

    Thượng sĩ

  • Thành viên
  • 265 Bài viết

Cho em hỏi bài này có bao nhiêu ẩn vậy ?

Mình quên chưa nói, có 3 ẩn x,y,z nhá :D


Ế là xu thế mang tầm cỡ quốc tế của các cấp bậc vai vế

 


#22
caybutbixanh

caybutbixanh

    Trung úy

  • Thành viên
  • 888 Bài viết

Bài 13 : Tìm tất cả các hàm số $f:\mathbb{R}\rightarrow \mathbb{R}$ sao cho 

$$x^{2}.f(x)+f(1-x)=2x-x^{4}, \forall x\in \mathbb{R}$$

 

Xin lưu ý : Lời giải trong TLCT chưa vét hết các nghiệm đâu nhé 


KẺ MẠNH CHƯA CHẮC ĐÃ THẮNG



MÀ KẺ THẮNG MỚI CHÍNH LÀ KẺ MẠNH!.



(FRANZ BECKEN BAUER)




ÔN THI MÔN HÓA HỌC TẠI ĐÂY.


#23
augustin louis cauchy

augustin louis cauchy

    Lính mới

  • Thành viên
  • 4 Bài viết

Cho tam giác ABC với a,b,c là độ dài 3 cạnh; r và R lần lượt là bán kính của đường tròn nội tiếp, ngoại tiếp tam giác ABC.

CMR $\left ( a-b \right )^{2}+\left ( b-c \right )^{2}+\left ( c-a \right )^{2}\leqslant 8R\left ( R-2r \right )$



#24
AnnieSally

AnnieSally

    Thiếu úy

  • Thành viên
  • 647 Bài viết

$\boxed{15}$ Trong mặt phẳng tọa độ cho một ngũ giác lồi có các đỉnh là những điểm có tọa độ nguyên. Chứng minh rằng bên trong hoặc trên cạnh ngũ giác có ít nhất một điểm có tọa độ nguyên


Bài viết đã được chỉnh sửa nội dung bởi AnnieSally: 08-10-2013 - 16:15


#25
LNH

LNH

    Bất Thế Tà Vương

  • Hiệp sỹ
  • 581 Bài viết

$\boxed{15}$ Trong mặt phẳng tọa độ cho một ngũ giác lồi có các đỉnh là những điểm có tọa độ nguyên. Chứng minh rằng bên trong hoặc trên cạnh ngũ giác có ít nhất một điểm có tọa độ nguyên

http://idoc.vn/tai-l...-lan-thu-8.html



#26
AnnieSally

AnnieSally

    Thiếu úy

  • Thành viên
  • 647 Bài viết

$\boxed{16}$ Hội Toán học của một thành phố cứ mỗi năm được nhóm họp 40 lần. Mỗi lần họp có đúng 10 thành viên đến dự, trong đó không có 2 thành viên nào cùng đến dự họp với nhau quá một lần.

Chứng minh rằng hội Toán học của thành phố không thể ít hơn 60 người.



#27
deathavailable

deathavailable

    Thượng sĩ

  • Thành viên
  • 265 Bài viết

$\boxed{16}$ Hội Toán học của một thành phố cứ mỗi năm được nhóm họp 40 lần. Mỗi lần họp có đúng 10 thành viên đến dự, trong đó không có 2 thành viên nào cùng đến dự họp với nhau quá một lần.

Chứng minh rằng hội Toán học của thành phố không thể ít hơn 60 người.

http://diendantoanho...-60-thành-viên/

 Ở đây nhé :D


Ế là xu thế mang tầm cỡ quốc tế của các cấp bậc vai vế

 


#28
AnnieSally

AnnieSally

    Thiếu úy

  • Thành viên
  • 647 Bài viết

Căn bản là đã biết làm và bây giờ chỉ cần một lời giải khác thui :)



#29
AnnieSally

AnnieSally

    Thiếu úy

  • Thành viên
  • 647 Bài viết

$\boxed{17}$ Cho tam giác $ABC$ và $x,y,z$ là các số dương

Tìm giá trị bé nhất của biểu thức sau: $P=\frac{1}{x}cos(A+\frac{B}{2})-\frac{1}{y}sin\frac{B}{2}-\frac{1}{z}sinC$



#30
AnnieSally

AnnieSally

    Thiếu úy

  • Thành viên
  • 647 Bài viết

$\boxed{18}$ Cho tam giác nhọn $ABC$ với các đường cao $AH$ $BK$ nội tiếp đường tròn $(O)$. Gọi $M$ là một điểm di động trên cung nhỏ $BC$ của đường tròn $(O)$ sao cho các đường thẳng $AM$ và $BK$ cắt nhau tại $E$; các đường thẳng $BM$ và $AH$ cắt nhau tại $F$. Chứng minh rằng khi $M$ di động trên cung nhỏ $BC$ của đường tròn $(O)$ thì trung điểm của đoạn $EF$ luôn nằm trên một đường thẳng cố định.


Bài viết đã được chỉnh sửa nội dung bởi AnnieSally: 08-10-2013 - 21:39


#31
Juliel

Juliel

    Thượng úy

  • Thành viên
  • 1240 Bài viết

Cho tam giác ABC với a,b,c là độ dài 3 cạnh; r và R lần lượt là bán kính của đường tròn nội tiếp, ngoại tiếp tam giác ABC.

CMR $\left ( a-b \right )^{2}+\left ( b-c \right )^{2}+\left ( c-a \right )^{2}\leqslant 8R\left ( R-2r \right )$

Khi nào m lên thì làm ơn bỏ chút thời gian quý báu để nhắn tin với gái giải cách khác giùm t cái.  :(

Lời giải :

Chú ý các đẳng thức $$ab+bc+ca=p^2+r^2+4Rr\;\;,\;\;a^{2}+b^{2}+c^{2}=(a+b+c)^{2}-2(ab+bc+ca)=4p^2-2(p^2+r^2+4Rr)=2p^2-2r^2-8Rr$$.

Viết lại BĐT :

$$a^{2}+b^{2}+c^{2}-ab-bc-ca\leq 4R^{2}-4Rr\Leftrightarrow (2p^2-2r^2-8Rr)-(p^2+r^2+4Rr)\leq 4R^{2}-4Rr\Leftrightarrow p^{2}\leq 4R^{2}+4Rr+3r^{2}$$

Đúng theo BĐT $Gerretsen$ thứ hai.


Bài viết đã được chỉnh sửa nội dung bởi Juliel: 19-10-2013 - 23:46

Đừng rời xa tôi vì tôi lỡ yêu người mất rồi !
 

Welcome to My Facebook !


#32
Juliel

Juliel

    Thượng úy

  • Thành viên
  • 1240 Bài viết

Bài 19 : Cho các số dương $a,b,c$ thỏa mãn $a^{2}+b^{2}+c^{2}+abc=4$. Chứng minh rằng :

$$a\sqrt{4-a^2}+a\sqrt{4-b^2}+a\sqrt{4-a^2}\leq 3\sqrt{3}$$


Bài viết đã được chỉnh sửa nội dung bởi Juliel: 20-10-2013 - 10:02

Đừng rời xa tôi vì tôi lỡ yêu người mất rồi !
 

Welcome to My Facebook !


#33
caybutbixanh

caybutbixanh

    Trung úy

  • Thành viên
  • 888 Bài viết

Bài 20 : Cho bốn số thực a,b,c,d thỏa mãn điều kiện $\left\{\begin{matrix} a^{2}+b^{2}=1\\ c+d=3 \end{matrix}\right.$

Chứng minh rằng  :$ac+bd+cd\leq \frac{9+6\sqrt{2}}{4}$

Bài 21: Cho các số thực dương $a,b,c$. Chứng minh rằng :

$\frac{\sqrt{a+b+c}+\sqrt{a}}{b+c}+\frac{\sqrt{a+b+c}+\sqrt{b}}{a+c}+\frac{\sqrt{a+b+c}+\sqrt{c}}{b+c}\geq \frac{9+3\sqrt{3}}{2\sqrt{a+b+c}}$

Bài 22 : Cho ba số $a,b,c$ thỏa mãn :$\left\{\begin{matrix} (a-b+c)^{2}+2ab-8ac+2bc< 0\\ 5a-4b+5c< 0 \end{matrix}\right.$

Chứng minh rằng :$2005a-2004b+2005c< 0$

--------------------------

Mọi người cho mình góp ý như thế này : Các bài toán post lên ở trong này nếu quá 2 ngày ( hoặc 3 ngày) mà chưa ai giải thì bạn post đề post luôn lời giải bài đó luôn nhá, tránh để topic bị loãng như mấy tháng nay :(.Các bài toán trong này khá khó nên sẽ có rất ít bạn giải cho nên mình thiết nghĩ nên vậy.

Vì một topic phát triển. :namtay

Thân.


KẺ MẠNH CHƯA CHẮC ĐÃ THẮNG



MÀ KẺ THẮNG MỚI CHÍNH LÀ KẺ MẠNH!.



(FRANZ BECKEN BAUER)




ÔN THI MÔN HÓA HỌC TẠI ĐÂY.


#34
shinichigl

shinichigl

    Trung sĩ

  • Thành viên
  • 135 Bài viết

cho 3 số a,b,c>0 và a+b+c=1, chứng minh

$\sqrt{a+bc}+\sqrt{b+ac}+\sqrt{c+ab}\geq 1+\sqrt{ab}+\sqrt{bc}+\sqrt{ac}$


Bài viết đã được chỉnh sửa nội dung bởi shinichigl: 06-01-2014 - 20:49

  • LNH yêu thích

#35
khong la gi ca

khong la gi ca

    Binh nhất

  • Thành viên
  • 33 Bài viết

$\boxed{12}$ Giải hệ phương trình :

$$\left\{\begin{matrix} x+y+z=0 (1) & & \\ \sqrt{x^{2}+xy+y^{2}}+\sqrt{y^{2}+yz+z^{2}}=\sqrt{z^{2}+zx+x^{2}} (2) & & \end{matrix}\right.$$

 

1. Xét trường hợp $y=0$
Khi đó hệ phương trình trở thành
$$\left\{\begin{matrix} x+z=0 & & \\ \left | x \right | + \left | z \right | + = \sqrt{z^{2}+zx+x^{2}} (a) & & \end{matrix}\right.$$

Vì hai vế của $(a)$ đều không âm nên

$$ (a) \Leftrightarrow x^2 + 2\left | x \right |\left | z \right | + z^2 = z^2 + zx + x^2 \Leftrightarrow 2\left | x \right |\left | z \right | = zx $$

Phương trình cuối cùng có nghiệm $x=0$ hoặc $z=0$, tuy nhiên cả hai nghiệm trên đều dẫn đến hệ sẽ có nghiệm $\left ( x,y,z \right ) = \left ( 0,0,0 \right )$

 

2. Trường hợp $y \neq 0$

Nhân hai vế của $(1)$ cho $y$ ta được

$$ xy + y^2 + yz = 0 $$

Thay vào $(2)$ cho ta

$$ \sqrt{x^{2}-yz}+\sqrt{z^{2}-xy}=\sqrt{z^{2}+zx+x^{2}} $$

Một lần nữa vì cả hai vế của phương trình trên đều không âm nên ta có

$$ x^{2}-yz + z^{2}-xy + 2 \sqrt{x^{2}-yz} \sqrt{z^{2}-xy} = z^{2}+zx+x^{2} $$

$$ \Leftrightarrow 2 \sqrt{x^{2}-yz} \sqrt{z^{2}-xy} = xy + yz + zx $$

 

Ta dễ dàng chứng minh được rằng

$$ xy + yz + zx \leq \frac{(a+b+c)^2}{3} = 0 $$

$$ 0 \leq 2 \sqrt{x^{2}-yz} \sqrt{z^{2}-xy} = xy + yz + zx $$

Theo đó ta phải có

$$ xy + yz + zx = 0 $$

suy ra $y^2 = zx$

 

Mặt khác $$ 2 \sqrt{x^{2}-yz} \sqrt{z^{2}-xy} = xy + yz + zx = 0 $$

Theo đó $x^2 = yz$ hoặc $z^2 = xy$. Ta sẽ chỉ xét trường hợp $x^2 = yz$, trường hợp còn lại được chứng minh tương tự.

Với $x^2 = yz$ thì

$$ y^2 = zx \Rightarrow y^4 = z^2 x^2 = z^3 y \Rightarrow z = y $$

Và hơn nữa lúc đó

$$ x^2 = y^2 \Rightarrow x = \pm y $$

Trong cả hai khả năng thì khi thay vào $(1)$ ta đều được $\left ( x,y,z \right ) = \left ( 0,0,0 \right )$ nhưng nghiệm này bị loại vì ta đang xét trường hợp $y \neq 0$.

 

Như vậy hệ phương trình chỉ có nghiệm duy nhất $\left ( x,y,z \right ) = \left ( 0,0,0 \right )$


"The Universe appears to be flawed.

If things exist because they ought to,

why are they not much better than they are?"


#36
phatsp

phatsp

    Binh nhất

  • Thành viên
  • 25 Bài viết

Cho hỏi là thi 30/4 co được sử dụng định lí stolz không? Hay chỉ cho phép sử dụng Cesaro thôi?



#37
Juliel

Juliel

    Thượng úy

  • Thành viên
  • 1240 Bài viết

Bài 21: Cho các số thực dương $a,b,c$. Chứng minh rằng :

$\frac{\sqrt{a+b+c}+\sqrt{a}}{b+c}+\frac{\sqrt{a+b+c}+\sqrt{b}}{a+c}+\frac{\sqrt{a+b+c}+\sqrt{c}}{b+c}\geq \frac{9+3\sqrt{3}}{2\sqrt{a+b+c}}$

Bất đẳng thức này là thuần nhất nên ta chuẩn hóa $a+b+c=1$. 

Bất đẳng thức cần chứng minh trở thành :

$$\dfrac{1+\sqrt{a}}{b+c}+\dfrac{1+\sqrt{b}}{c+a}+\dfrac{1+\sqrt{c}}{a+b}\geq \dfrac{9+3\sqrt{3}}{2}$$

Ta đổi biến $a=x^2,b=y^2,c=z^2$ thì $x^2+y^2+z^2=1$ và cần chứng minh :

$$\dfrac{1}{x^2+y^2}+\dfrac{1}{y^2+z^2}+\dfrac{1}{z^2+x^2}+\dfrac{x}{y^2+z^2}+\dfrac{y}{z^2+x^2}+\dfrac{z}{x^2+y^2}\geq \dfrac{9+3\sqrt{3}}{2}$$

Thật vậy, theo $AM-GM$

$$x(1-x^2)=x(1-x)(1+x)=\dfrac{1}{(2-\sqrt{3})(\sqrt{3}-1)}.(1-x)\left [ (2-\sqrt{3})+(2-\sqrt{3})x \right ].(\sqrt{3}-1)x\leq \dfrac{1}{(2-\sqrt{3})(\sqrt{3}-1)}.\left [ \dfrac{(1-x)+(2-\sqrt{3})+(2-\sqrt{3})x+(\sqrt{3}-1)x}{3} \right ]^3=\dfrac{2\sqrt{3}}{9}\Leftrightarrow \dfrac{x}{1-x^2}\geq \dfrac{3\sqrt{3}x^2}{2}\Leftrightarrow \dfrac{x}{y^2+z^2}\geq \dfrac{3\sqrt{3}x^2}{2}$$

Kéo theo :

$$\dfrac{x}{y^2+z^2}+\dfrac{y}{z^2+x^2}+\dfrac{z}{x^2+y^2}\geq \dfrac{3\sqrt{3}}{2}(x^2+y^2+z^2)=\dfrac{3\sqrt{3}}{2}\;\;\;\;(1)$$

Hơn nữa theo $Cauchy-Schwarz$ ta còn có :

$$\dfrac{1}{y^2+z^2}+\dfrac{1}{z^2+x^2}+\dfrac{1}{x^2+y^2}\geq \dfrac{9}{2(x^2+y^2+z^2)}=\dfrac{9}{2}\;\;\;(2)$$

Cộng vế theo vế $(1)(2)$ ta được điều phải chứng minh.


Đừng rời xa tôi vì tôi lỡ yêu người mất rồi !
 

Welcome to My Facebook !


#38
Juliel

Juliel

    Thượng úy

  • Thành viên
  • 1240 Bài viết

Bài 20 : Cho bốn số thực a,b,c,d thỏa mãn điều kiện $\left\{\begin{matrix} a^{2}+b^{2}=1\\ c+d=3 \end{matrix}\right.$

Chứng minh rằng  :$ac+bd+cd\leq \frac{9+6\sqrt{2}}{4}$

Thay $d=3-c$ ta viết biểu thức vế trái thành một tam thức bậc hai ẩn $c$ :

$S=ac+b(3-c)+c(3-c)=-c^2-c(b-a-3)+3b$

Sử dụng tính chất $Ax^2+Bx+C\leq \dfrac{4AC-B^2}{4A}$ ta được :

$S=\leq \dfrac{-12b-(b-a-3)^2}{-4}=\dfrac{12b+(a-b+3)^2}{4}$

Vậy cần chỉ ra rằng

$\dfrac{12b+(a-b+3)^2}{4}\leq \dfrac{9+6\sqrt{2}}{4}\Leftrightarrow 12b+(a-b)^2+6(a-b)+9\leq 9+6\sqrt{2}\Leftrightarrow (a^2+b^2)+6(a+b)-2ab\leq 6\sqrt{2}\Leftrightarrow 6(a+b)-2ab\leq 6\sqrt{2}-1$

Mà $2ab=(a+b)^2-a^2-b^2=(a+b)^2-1$ nên chứng minh :

$l-(a+b)^2+6(a+b)+2-6\sqrt{2}\leq 0\Leftrightarrow a+b\leq \sqrt{2}\;\vee \;a+b\geq 6-\sqrt{2}$

Hiển nhiên vì $a+b\leq \sqrt{2(a^2+b^2)}=\sqrt{2}$

Ta có điều phải chứng minh.


Bài viết đã được chỉnh sửa nội dung bởi Juliel: 28-03-2014 - 18:13

Đừng rời xa tôi vì tôi lỡ yêu người mất rồi !
 

Welcome to My Facebook !


#39
Juliel

Juliel

    Thượng úy

  • Thành viên
  • 1240 Bài viết

cho 3 số $a,b,c>0$ và $a+b+c=$1, chứng minh

$\sqrt{a+bc}+\sqrt{b+ac}+\sqrt{c+ab}\geq 1+\sqrt{ab}+\sqrt{bc}+\sqrt{ac}$

Áp dụng BĐT $Cauchy-Schwarz$ :

$$\sqrt{a+bc}=\sqrt{a(a+b+c)+bc}=\sqrt{(a+c)(a+b)}\geq a+\sqrt{bc}$$

Thiết lập các BĐT tương tự và cuối cùng ta được :

$$\sqrt{a+bc}+\sqrt{b+ac}+\sqrt{c+ab}\geq a+b+c+\sqrt{ab}+\sqrt{bc}+\sqrt{ac}=1+\sqrt{ab}+\sqrt{bc}+\sqrt{ac}$$

Đây là điều cần chứng minh.

 

Nếu ta thay $a,b,c$ lần lượt bởi $1/a,1/b,1/c$ ta được bài APMO 2002


Đừng rời xa tôi vì tôi lỡ yêu người mất rồi !
 

Welcome to My Facebook !


#40
Juliel

Juliel

    Thượng úy

  • Thành viên
  • 1240 Bài viết

Bài 23 : Tìm tất cả các hàm số $f:\mathbb{Q}^+\rightarrow \mathbb{Q}^+$ và thỏa mãn :

$$\left\{\begin{matrix} f(x+1)=f(x)+1\\ f(x^5)=f^5(x)\\ \end{matrix}\right.,\;\forall x\in \mathbb{Q}^+$$

 

Bài 24 : Giải phương trình :

$$x=\sqrt[5]{11\sqrt[5]{11\sqrt[5]{11x+10}+10}+10}$$


Đừng rời xa tôi vì tôi lỡ yêu người mất rồi !
 

Welcome to My Facebook !





0 người đang xem chủ đề

0 thành viên, 0 khách, 0 thành viên ẩn danh